Assume $p, q$ are prime numbers. Decide whether it is possible for $12p + 12q$ to be equal to$ pq$ .How does law of quadratic reciprocity work?Is this an acceptable congruency proof?Prove that every non-prime natural number $ > 1$ can be written in the form of $n+(n+2)+(n+4)+…+(n+2m) = p$Positive integers $n$ which can be written as $x^2-3y^2$Show that $p^3+4$ is primePlease verify my induction proof.Divisibility conditionFor any natural numbers a, b, c, d if a*b = c*d is it possible that a + b + c + d is prime numberA question about proving distinct Fermat numbers are relatively primeFind $p, q, r$ prime numbers such that $r^2 - q^2 - p^2 = n^2$.

Brexit - No Deal Rejection

How to generate globally unique ids for different tables of the same database?

Why doesn't the EU now just force the UK to choose between referendum and no-deal?

PlotLabels with equations not expressions

Old race car problem/puzzle

Meaning of "SEVERA INDEOVI VAS" from 3rd Century slab

Sword in the Stone story where the sword was held in place by electromagnets

Good allowance savings plan?

Am I not good enough for you?

In-house repeater?

Counting certain elements in lists

Why do Australian milk farmers need to protest supermarkets' milk price?

Is it normal that my co-workers at a fitness company criticize my food choices?

Why is "das Weib" grammatically neuter?

Happy pi day, everyone!

How is the Swiss post e-voting system supposed to work, and how was it wrong?

Did CPM support custom hardware using device drivers?

Could the Saturn V actually have launched astronauts around Venus?

How to answer questions about my characters?

Official degrees of earth’s rotation per day

What are the possible solutions of the given equation?

PTIJ: Who should pay for Uber rides: the child or the parent?

Life insurance that covers only simultaneous/dual deaths

What options are left, if Britain cannot decide?



Assume $p, q$ are prime numbers. Decide whether it is possible for $12p + 12q$ to be equal to$ pq$ .


How does law of quadratic reciprocity work?Is this an acceptable congruency proof?Prove that every non-prime natural number $ > 1$ can be written in the form of $n+(n+2)+(n+4)+…+(n+2m) = p$Positive integers $n$ which can be written as $x^2-3y^2$Show that $p^3+4$ is primePlease verify my induction proof.Divisibility conditionFor any natural numbers a, b, c, d if a*b = c*d is it possible that a + b + c + d is prime numberA question about proving distinct Fermat numbers are relatively primeFind $p, q, r$ prime numbers such that $r^2 - q^2 - p^2 = n^2$.













1












$begingroup$


So far I have managed the following



$2(6p + 6q) = pq$



$p$ and $q$ cannot both be even as the only prime number that is even is $2$ and that does not prove this equation.



$p$ and $q$ cannot both be odd as $2$ * anything is even



So that means that one of $p$, $q$ must be odd and the other must be even.



So let $p = 2k$ and $q = 2L+1$



$2(12k + 12L + 6) = (2k) * (2L + 1) = 24k + 24L + 12 = 4kL + 2k$



Have I done this correctly and do I have sufficient proof to claim that $12p + 12q$ cannot be equal to $pq$ given that both $p$ and $q$ are both prime numbers?










share|cite|improve this question









New contributor




Akaion is a new contributor to this site. Take care in asking for clarification, commenting, and answering.
Check out our Code of Conduct.







$endgroup$
















    1












    $begingroup$


    So far I have managed the following



    $2(6p + 6q) = pq$



    $p$ and $q$ cannot both be even as the only prime number that is even is $2$ and that does not prove this equation.



    $p$ and $q$ cannot both be odd as $2$ * anything is even



    So that means that one of $p$, $q$ must be odd and the other must be even.



    So let $p = 2k$ and $q = 2L+1$



    $2(12k + 12L + 6) = (2k) * (2L + 1) = 24k + 24L + 12 = 4kL + 2k$



    Have I done this correctly and do I have sufficient proof to claim that $12p + 12q$ cannot be equal to $pq$ given that both $p$ and $q$ are both prime numbers?










    share|cite|improve this question









    New contributor




    Akaion is a new contributor to this site. Take care in asking for clarification, commenting, and answering.
    Check out our Code of Conduct.







    $endgroup$














      1












      1








      1





      $begingroup$


      So far I have managed the following



      $2(6p + 6q) = pq$



      $p$ and $q$ cannot both be even as the only prime number that is even is $2$ and that does not prove this equation.



      $p$ and $q$ cannot both be odd as $2$ * anything is even



      So that means that one of $p$, $q$ must be odd and the other must be even.



      So let $p = 2k$ and $q = 2L+1$



      $2(12k + 12L + 6) = (2k) * (2L + 1) = 24k + 24L + 12 = 4kL + 2k$



      Have I done this correctly and do I have sufficient proof to claim that $12p + 12q$ cannot be equal to $pq$ given that both $p$ and $q$ are both prime numbers?










      share|cite|improve this question









      New contributor




      Akaion is a new contributor to this site. Take care in asking for clarification, commenting, and answering.
      Check out our Code of Conduct.







      $endgroup$




      So far I have managed the following



      $2(6p + 6q) = pq$



      $p$ and $q$ cannot both be even as the only prime number that is even is $2$ and that does not prove this equation.



      $p$ and $q$ cannot both be odd as $2$ * anything is even



      So that means that one of $p$, $q$ must be odd and the other must be even.



      So let $p = 2k$ and $q = 2L+1$



      $2(12k + 12L + 6) = (2k) * (2L + 1) = 24k + 24L + 12 = 4kL + 2k$



      Have I done this correctly and do I have sufficient proof to claim that $12p + 12q$ cannot be equal to $pq$ given that both $p$ and $q$ are both prime numbers?







      elementary-number-theory






      share|cite|improve this question









      New contributor




      Akaion is a new contributor to this site. Take care in asking for clarification, commenting, and answering.
      Check out our Code of Conduct.











      share|cite|improve this question









      New contributor




      Akaion is a new contributor to this site. Take care in asking for clarification, commenting, and answering.
      Check out our Code of Conduct.









      share|cite|improve this question




      share|cite|improve this question








      edited Mar 11 at 9:16









      dmtri

      1,6172521




      1,6172521






      New contributor




      Akaion is a new contributor to this site. Take care in asking for clarification, commenting, and answering.
      Check out our Code of Conduct.









      asked Mar 11 at 8:25









      AkaionAkaion

      203




      203




      New contributor




      Akaion is a new contributor to this site. Take care in asking for clarification, commenting, and answering.
      Check out our Code of Conduct.





      New contributor





      Akaion is a new contributor to this site. Take care in asking for clarification, commenting, and answering.
      Check out our Code of Conduct.






      Akaion is a new contributor to this site. Take care in asking for clarification, commenting, and answering.
      Check out our Code of Conduct.




















          3 Answers
          3






          active

          oldest

          votes


















          3












          $begingroup$

          While a good answer has been given, and your own calculations are not wrong, they are just short of the goal, as it isn't apparent why your last result



          $$24k+24L+12=4kL+2k$$



          is any kind of contradiction. That only becomes clear once you realize that you set $p=2k$, and with $p$ being a prime, $k=1$ must follow. Then your last result becomes



          $$24+24L+12=4L+2$$



          and this is clearly impossible. One argument is that the left hand side is clearly bigger then the right hand side, another would be that the left hand side is divisible by 4 while the right hand side isn't.



          To improve your approach for readability, I suggest just mentioning that $p$ and $q$ can't both be odd first (as you did), this means w.l.o.g $p=2$. Then the equation becomes
          $24+12q=2q$, which is impossible due to the "left hand side bigger then right hand side" approach. No need to introduce $k,L$.






          share|cite|improve this answer









          $endgroup$




















            4












            $begingroup$

            It is obviously false because $pq$ has only two prime factors, but $12(p+q)$ has at least $4$ prime factors.






            share|cite|improve this answer









            $endgroup$












            • $begingroup$
              You mean proper divisors or prime factors? If $p=2, q=3$, then $12(p+q)$ has only $3$ distinct prime factors, though of course it has at least $4$ proper divisors.
              $endgroup$
              – Macavity
              Mar 11 at 8:34










            • $begingroup$
              I didn't say distinct prime factors, I said prime factors. $6$ has two prime factors whereas $60$ has four prime factors.
              $endgroup$
              – Peter Foreman
              Mar 11 at 8:36


















            0












            $begingroup$

            0) Let $p,q$ be prime numbers.



            1) $p,q not =2$.



            Then $pq$ is odd, while $12(p+q)$ is even, ruled out.



            2) Assume $p=q=2$.



            Then $pq =4$, ruled out (why?)



            3)Assume $p=2$, $q not =2$.



            Then $q= 6(p+q)$, not a prime , ruled out.



            Hence?






            share|cite|improve this answer









            $endgroup$












              Your Answer





              StackExchange.ifUsing("editor", function ()
              return StackExchange.using("mathjaxEditing", function ()
              StackExchange.MarkdownEditor.creationCallbacks.add(function (editor, postfix)
              StackExchange.mathjaxEditing.prepareWmdForMathJax(editor, postfix, [["$", "$"], ["\\(","\\)"]]);
              );
              );
              , "mathjax-editing");

              StackExchange.ready(function()
              var channelOptions =
              tags: "".split(" "),
              id: "69"
              ;
              initTagRenderer("".split(" "), "".split(" "), channelOptions);

              StackExchange.using("externalEditor", function()
              // Have to fire editor after snippets, if snippets enabled
              if (StackExchange.settings.snippets.snippetsEnabled)
              StackExchange.using("snippets", function()
              createEditor();
              );

              else
              createEditor();

              );

              function createEditor()
              StackExchange.prepareEditor(
              heartbeatType: 'answer',
              autoActivateHeartbeat: false,
              convertImagesToLinks: true,
              noModals: true,
              showLowRepImageUploadWarning: true,
              reputationToPostImages: 10,
              bindNavPrevention: true,
              postfix: "",
              imageUploader:
              brandingHtml: "Powered by u003ca class="icon-imgur-white" href="https://imgur.com/"u003eu003c/au003e",
              contentPolicyHtml: "User contributions licensed under u003ca href="https://creativecommons.org/licenses/by-sa/3.0/"u003ecc by-sa 3.0 with attribution requiredu003c/au003e u003ca href="https://stackoverflow.com/legal/content-policy"u003e(content policy)u003c/au003e",
              allowUrls: true
              ,
              noCode: true, onDemand: true,
              discardSelector: ".discard-answer"
              ,immediatelyShowMarkdownHelp:true
              );



              );






              Akaion is a new contributor. Be nice, and check out our Code of Conduct.









              draft saved

              draft discarded


















              StackExchange.ready(
              function ()
              StackExchange.openid.initPostLogin('.new-post-login', 'https%3a%2f%2fmath.stackexchange.com%2fquestions%2f3143434%2fassume-p-q-are-prime-numbers-decide-whether-it-is-possible-for-12p-12q-t%23new-answer', 'question_page');

              );

              Post as a guest















              Required, but never shown

























              3 Answers
              3






              active

              oldest

              votes








              3 Answers
              3






              active

              oldest

              votes









              active

              oldest

              votes






              active

              oldest

              votes









              3












              $begingroup$

              While a good answer has been given, and your own calculations are not wrong, they are just short of the goal, as it isn't apparent why your last result



              $$24k+24L+12=4kL+2k$$



              is any kind of contradiction. That only becomes clear once you realize that you set $p=2k$, and with $p$ being a prime, $k=1$ must follow. Then your last result becomes



              $$24+24L+12=4L+2$$



              and this is clearly impossible. One argument is that the left hand side is clearly bigger then the right hand side, another would be that the left hand side is divisible by 4 while the right hand side isn't.



              To improve your approach for readability, I suggest just mentioning that $p$ and $q$ can't both be odd first (as you did), this means w.l.o.g $p=2$. Then the equation becomes
              $24+12q=2q$, which is impossible due to the "left hand side bigger then right hand side" approach. No need to introduce $k,L$.






              share|cite|improve this answer









              $endgroup$

















                3












                $begingroup$

                While a good answer has been given, and your own calculations are not wrong, they are just short of the goal, as it isn't apparent why your last result



                $$24k+24L+12=4kL+2k$$



                is any kind of contradiction. That only becomes clear once you realize that you set $p=2k$, and with $p$ being a prime, $k=1$ must follow. Then your last result becomes



                $$24+24L+12=4L+2$$



                and this is clearly impossible. One argument is that the left hand side is clearly bigger then the right hand side, another would be that the left hand side is divisible by 4 while the right hand side isn't.



                To improve your approach for readability, I suggest just mentioning that $p$ and $q$ can't both be odd first (as you did), this means w.l.o.g $p=2$. Then the equation becomes
                $24+12q=2q$, which is impossible due to the "left hand side bigger then right hand side" approach. No need to introduce $k,L$.






                share|cite|improve this answer









                $endgroup$















                  3












                  3








                  3





                  $begingroup$

                  While a good answer has been given, and your own calculations are not wrong, they are just short of the goal, as it isn't apparent why your last result



                  $$24k+24L+12=4kL+2k$$



                  is any kind of contradiction. That only becomes clear once you realize that you set $p=2k$, and with $p$ being a prime, $k=1$ must follow. Then your last result becomes



                  $$24+24L+12=4L+2$$



                  and this is clearly impossible. One argument is that the left hand side is clearly bigger then the right hand side, another would be that the left hand side is divisible by 4 while the right hand side isn't.



                  To improve your approach for readability, I suggest just mentioning that $p$ and $q$ can't both be odd first (as you did), this means w.l.o.g $p=2$. Then the equation becomes
                  $24+12q=2q$, which is impossible due to the "left hand side bigger then right hand side" approach. No need to introduce $k,L$.






                  share|cite|improve this answer









                  $endgroup$



                  While a good answer has been given, and your own calculations are not wrong, they are just short of the goal, as it isn't apparent why your last result



                  $$24k+24L+12=4kL+2k$$



                  is any kind of contradiction. That only becomes clear once you realize that you set $p=2k$, and with $p$ being a prime, $k=1$ must follow. Then your last result becomes



                  $$24+24L+12=4L+2$$



                  and this is clearly impossible. One argument is that the left hand side is clearly bigger then the right hand side, another would be that the left hand side is divisible by 4 while the right hand side isn't.



                  To improve your approach for readability, I suggest just mentioning that $p$ and $q$ can't both be odd first (as you did), this means w.l.o.g $p=2$. Then the equation becomes
                  $24+12q=2q$, which is impossible due to the "left hand side bigger then right hand side" approach. No need to introduce $k,L$.







                  share|cite|improve this answer












                  share|cite|improve this answer



                  share|cite|improve this answer










                  answered Mar 11 at 8:38









                  IngixIngix

                  4,857159




                  4,857159





















                      4












                      $begingroup$

                      It is obviously false because $pq$ has only two prime factors, but $12(p+q)$ has at least $4$ prime factors.






                      share|cite|improve this answer









                      $endgroup$












                      • $begingroup$
                        You mean proper divisors or prime factors? If $p=2, q=3$, then $12(p+q)$ has only $3$ distinct prime factors, though of course it has at least $4$ proper divisors.
                        $endgroup$
                        – Macavity
                        Mar 11 at 8:34










                      • $begingroup$
                        I didn't say distinct prime factors, I said prime factors. $6$ has two prime factors whereas $60$ has four prime factors.
                        $endgroup$
                        – Peter Foreman
                        Mar 11 at 8:36















                      4












                      $begingroup$

                      It is obviously false because $pq$ has only two prime factors, but $12(p+q)$ has at least $4$ prime factors.






                      share|cite|improve this answer









                      $endgroup$












                      • $begingroup$
                        You mean proper divisors or prime factors? If $p=2, q=3$, then $12(p+q)$ has only $3$ distinct prime factors, though of course it has at least $4$ proper divisors.
                        $endgroup$
                        – Macavity
                        Mar 11 at 8:34










                      • $begingroup$
                        I didn't say distinct prime factors, I said prime factors. $6$ has two prime factors whereas $60$ has four prime factors.
                        $endgroup$
                        – Peter Foreman
                        Mar 11 at 8:36













                      4












                      4








                      4





                      $begingroup$

                      It is obviously false because $pq$ has only two prime factors, but $12(p+q)$ has at least $4$ prime factors.






                      share|cite|improve this answer









                      $endgroup$



                      It is obviously false because $pq$ has only two prime factors, but $12(p+q)$ has at least $4$ prime factors.







                      share|cite|improve this answer












                      share|cite|improve this answer



                      share|cite|improve this answer










                      answered Mar 11 at 8:27









                      Peter ForemanPeter Foreman

                      3,7921216




                      3,7921216











                      • $begingroup$
                        You mean proper divisors or prime factors? If $p=2, q=3$, then $12(p+q)$ has only $3$ distinct prime factors, though of course it has at least $4$ proper divisors.
                        $endgroup$
                        – Macavity
                        Mar 11 at 8:34










                      • $begingroup$
                        I didn't say distinct prime factors, I said prime factors. $6$ has two prime factors whereas $60$ has four prime factors.
                        $endgroup$
                        – Peter Foreman
                        Mar 11 at 8:36
















                      • $begingroup$
                        You mean proper divisors or prime factors? If $p=2, q=3$, then $12(p+q)$ has only $3$ distinct prime factors, though of course it has at least $4$ proper divisors.
                        $endgroup$
                        – Macavity
                        Mar 11 at 8:34










                      • $begingroup$
                        I didn't say distinct prime factors, I said prime factors. $6$ has two prime factors whereas $60$ has four prime factors.
                        $endgroup$
                        – Peter Foreman
                        Mar 11 at 8:36















                      $begingroup$
                      You mean proper divisors or prime factors? If $p=2, q=3$, then $12(p+q)$ has only $3$ distinct prime factors, though of course it has at least $4$ proper divisors.
                      $endgroup$
                      – Macavity
                      Mar 11 at 8:34




                      $begingroup$
                      You mean proper divisors or prime factors? If $p=2, q=3$, then $12(p+q)$ has only $3$ distinct prime factors, though of course it has at least $4$ proper divisors.
                      $endgroup$
                      – Macavity
                      Mar 11 at 8:34












                      $begingroup$
                      I didn't say distinct prime factors, I said prime factors. $6$ has two prime factors whereas $60$ has four prime factors.
                      $endgroup$
                      – Peter Foreman
                      Mar 11 at 8:36




                      $begingroup$
                      I didn't say distinct prime factors, I said prime factors. $6$ has two prime factors whereas $60$ has four prime factors.
                      $endgroup$
                      – Peter Foreman
                      Mar 11 at 8:36











                      0












                      $begingroup$

                      0) Let $p,q$ be prime numbers.



                      1) $p,q not =2$.



                      Then $pq$ is odd, while $12(p+q)$ is even, ruled out.



                      2) Assume $p=q=2$.



                      Then $pq =4$, ruled out (why?)



                      3)Assume $p=2$, $q not =2$.



                      Then $q= 6(p+q)$, not a prime , ruled out.



                      Hence?






                      share|cite|improve this answer









                      $endgroup$

















                        0












                        $begingroup$

                        0) Let $p,q$ be prime numbers.



                        1) $p,q not =2$.



                        Then $pq$ is odd, while $12(p+q)$ is even, ruled out.



                        2) Assume $p=q=2$.



                        Then $pq =4$, ruled out (why?)



                        3)Assume $p=2$, $q not =2$.



                        Then $q= 6(p+q)$, not a prime , ruled out.



                        Hence?






                        share|cite|improve this answer









                        $endgroup$















                          0












                          0








                          0





                          $begingroup$

                          0) Let $p,q$ be prime numbers.



                          1) $p,q not =2$.



                          Then $pq$ is odd, while $12(p+q)$ is even, ruled out.



                          2) Assume $p=q=2$.



                          Then $pq =4$, ruled out (why?)



                          3)Assume $p=2$, $q not =2$.



                          Then $q= 6(p+q)$, not a prime , ruled out.



                          Hence?






                          share|cite|improve this answer









                          $endgroup$



                          0) Let $p,q$ be prime numbers.



                          1) $p,q not =2$.



                          Then $pq$ is odd, while $12(p+q)$ is even, ruled out.



                          2) Assume $p=q=2$.



                          Then $pq =4$, ruled out (why?)



                          3)Assume $p=2$, $q not =2$.



                          Then $q= 6(p+q)$, not a prime , ruled out.



                          Hence?







                          share|cite|improve this answer












                          share|cite|improve this answer



                          share|cite|improve this answer










                          answered Mar 11 at 9:25









                          Peter SzilasPeter Szilas

                          11.5k2822




                          11.5k2822




















                              Akaion is a new contributor. Be nice, and check out our Code of Conduct.









                              draft saved

                              draft discarded


















                              Akaion is a new contributor. Be nice, and check out our Code of Conduct.












                              Akaion is a new contributor. Be nice, and check out our Code of Conduct.











                              Akaion is a new contributor. Be nice, and check out our Code of Conduct.














                              Thanks for contributing an answer to Mathematics Stack Exchange!


                              • Please be sure to answer the question. Provide details and share your research!

                              But avoid


                              • Asking for help, clarification, or responding to other answers.

                              • Making statements based on opinion; back them up with references or personal experience.

                              Use MathJax to format equations. MathJax reference.


                              To learn more, see our tips on writing great answers.




                              draft saved


                              draft discarded














                              StackExchange.ready(
                              function ()
                              StackExchange.openid.initPostLogin('.new-post-login', 'https%3a%2f%2fmath.stackexchange.com%2fquestions%2f3143434%2fassume-p-q-are-prime-numbers-decide-whether-it-is-possible-for-12p-12q-t%23new-answer', 'question_page');

                              );

                              Post as a guest















                              Required, but never shown





















































                              Required, but never shown














                              Required, but never shown












                              Required, but never shown







                              Required, but never shown

































                              Required, but never shown














                              Required, but never shown












                              Required, but never shown







                              Required, but never shown







                              Popular posts from this blog

                              Lowndes Grove History Architecture References Navigation menu32°48′6″N 79°57′58″W / 32.80167°N 79.96611°W / 32.80167; -79.9661132°48′6″N 79°57′58″W / 32.80167°N 79.96611°W / 32.80167; -79.9661178002500"National Register Information System"Historic houses of South Carolina"Lowndes Grove""+32° 48' 6.00", −79° 57' 58.00""Lowndes Grove, Charleston County (260 St. Margaret St., Charleston)""Lowndes Grove"The Charleston ExpositionIt Happened in South Carolina"Lowndes Grove (House), Saint Margaret Street & Sixth Avenue, Charleston, Charleston County, SC(Photographs)"Plantations of the Carolina Low Countrye

                              random experiment with two different functions on unit interval Announcing the arrival of Valued Associate #679: Cesar Manara Planned maintenance scheduled April 23, 2019 at 00:00UTC (8:00pm US/Eastern)Random variable and probability space notionsRandom Walk with EdgesFinding functions where the increase over a random interval is Poisson distributedNumber of days until dayCan an observed event in fact be of zero probability?Unit random processmodels of coins and uniform distributionHow to get the number of successes given $n$ trials , probability $P$ and a random variable $X$Absorbing Markov chain in a computer. Is “almost every” turned into always convergence in computer executions?Stopped random walk is not uniformly integrable

                              How should I support this large drywall patch? Planned maintenance scheduled April 23, 2019 at 00:00UTC (8:00pm US/Eastern) Announcing the arrival of Valued Associate #679: Cesar Manara Unicorn Meta Zoo #1: Why another podcast?How do I cover large gaps in drywall?How do I keep drywall around a patch from crumbling?Can I glue a second layer of drywall?How to patch long strip on drywall?Large drywall patch: how to avoid bulging seams?Drywall Mesh Patch vs. Bulge? To remove or not to remove?How to fix this drywall job?Prep drywall before backsplashWhat's the best way to fix this horrible drywall patch job?Drywall patching using 3M Patch Plus Primer